Những câu hỏi liên quan
Dương Thị Như Quỳnh
Xem chi tiết
ST
8 tháng 7 2018 lúc 14:27

Ta có: \(\frac{1}{2^2}< \frac{1}{1.2};\frac{1}{3^2}< \frac{1}{2.3};...;\frac{1}{n^2}< \frac{1}{\left(n-1\right)n}\)

\(\Rightarrow S< \frac{1}{1.2}+\frac{1}{2.3}+...+\frac{1}{\left(n-1\right)n}=1-\frac{1}{2}+\frac{1}{2}-\frac{1}{3}+...+\frac{1}{n-1}-\frac{1}{n}=1-\frac{1}{n}< 1\)

Vậy S<1

Bình luận (0)
Arima Kousei
8 tháng 7 2018 lúc 14:27

Ta có :

\(\frac{1}{2^2}+\frac{1}{3^2}+\frac{1}{4^2}+...+\frac{1}{\left(n-1\right)^2}+\frac{1}{n^2}< \frac{1}{1.2}+\frac{1}{2.3}+\frac{1}{3.4}+...+\frac{1}{\left(n-2\right)\left(n-1\right)}+\frac{1}{\left(n-1\right)n}\)

\(\Rightarrow S< 1-\frac{1}{2}+\frac{1}{2}-\frac{1}{3}+\frac{1}{3}-\frac{1}{4}+...+\frac{1}{n-2}-\frac{1}{n-1}+\frac{1}{n-1}-\frac{1}{n}\)

\(\Rightarrow S< 1-\frac{1}{n}< 1\)

Vậy \(S=1\)

Bình luận (0)
Nguyễn Khắc Thành
Xem chi tiết
Yuki_Kali_Ruby
Xem chi tiết
Nguyễn Trần Tú kiz
Xem chi tiết
Nguyễn Anh Quân
21 tháng 1 2018 lúc 20:44

Có : S = (1+2)+(2^2+2^3)+.....+(2^98+2^99)

= 3+2^2.(1+2)+......+2^98.(1+2)

= 3+2^2.3+.....+2^98.3

= 3.(1+2^2+......+2^98) chia hết cho 3

=> S chia hết cho 3

Có : 2S = 2+2^2+....+2^100

S = 2S - S = (2+2^2+....+2^100)-(1+2+2^2+....+2^99) = 2^100 - 1

=> S+1 = 2^100-1+1 = 2^100 = (2^2)^50 = 4^50 = 4^48+2

=> ĐPCM

Tk mk nha

Bình luận (0)
Nguyễn Trần Tú kiz
21 tháng 1 2018 lúc 20:47

Cảm Ơn Bạn Nhiều!

Bình luận (0)
Mai Phương Nguyễn
Xem chi tiết
Nguyễn Hoàng Minh
11 tháng 12 2021 lúc 21:37

\(S=\left(1-\dfrac{1}{4}\right)+\left(1-\dfrac{1}{9}\right)+\left(1-\dfrac{1}{16}\right)+...+\left(1-\dfrac{1}{n^2}\right)\\ S=\left(1+1+...+1\right)-\left(\dfrac{1}{4}+\dfrac{1}{9}+...+\dfrac{1}{n^2}\right)\\ S=n-1-\left(\dfrac{1}{4}+\dfrac{1}{9}+...+\dfrac{1}{n^2}\right)< n-1\)

Lại có \(\dfrac{1}{4}+\dfrac{1}{9}+..+\dfrac{1}{n^2}=\dfrac{1}{2^2}+\dfrac{1}{3^2}+...+\dfrac{1}{n^2}\)

\(\Rightarrow\dfrac{1}{4}+\dfrac{1}{9}+...+\dfrac{1}{n^2}< \dfrac{1}{1.2}+\dfrac{1}{2.3}+...+\dfrac{1}{n\left(n-1\right)}< 1-\dfrac{1}{2}+\dfrac{1}{2}-\dfrac{1}{3}+...+\dfrac{1}{n-1}-\dfrac{1}{n}=1-\dfrac{1}{n}< 1\)

\(\Rightarrow S>n-1-1=n-2\\ \Rightarrow n-2< S< n-1\\ \Rightarrow S\notin N\)

Bình luận (0)
Phuc Thao
Xem chi tiết
Nguyễn Thị Thương Hoài
5 tháng 12 2023 lúc 15:25

                          Bài 1: 

   (1 - 2 + 3 - 4+ ... - 96 + 97 - 98 + 99).\(x\) = 2000

Đặt A = 1 - 2 + 3  - 4 +...- 96 + 97 - 98 + 99 

Xét dãy số: 1; 2; 3; 4;...;96; 97; 98; 99

Dãy số trên là dãy số cách đều với khoảng cách là: 2 - 1 = 1

Số số hạng của dãy số trên là: (99 - 1): 1 +  = 99

                  Vì 99 : 2 = 49 dư 1

Nhóm 2 số hạng liên tiếp của A thành một nhóm thì A là tổng của 49 nhóm và 99

A = 1 - 2 + 3  - 4 + ... - 96 + 97 - 98 + 99

A = (1- 2) + (3 - 4)+ ...+ (97 - 98) + 99

A =   - 1 + (-1) + (-1) +...+ (-1) + 99

A = -1.49 + 99

A = -49 + 99

A = 50 Thay A = 

Vậy 50.\(x\) = 2000

            \(x\) = 2000 : 50

             \(x\) = 40

       

 

 

           

 

      

Bình luận (0)
Nguyễn Thị Thương Hoài
5 tháng 12 2023 lúc 15:28

2, n và n + 1

Gọi ước chung lớn nhất của n và n + 1 là d

Ta có: n ⋮ d;  n + 1 ⋮ d 

⇒ n + 1  - n ⋮ d 

                1 ⋮ d

                d = 1

Vậy ƯCLN(n +1; n) = 1 Hay  n + 1; n là hai số nguyên tố cùng nhau (đpcm)

 

Bình luận (0)
Nguyễn Thị Thương Hoài
5 tháng 12 2023 lúc 15:32

b, 2n và 2n + 3 là hai số nguyên tố cùng nhau

    Gọi ƯCLN( 2n; 2n + 3) = d

⇒ 2n ⋮ d; 2n + 3 ⋮ d

⇒ 2n + 3  - 2n ⋮ d

            3         ⋮  d

           d = 1; 3

2n và 2n + 3 không thể là hai số cùng nhau

Bình luận (0)
Nguyen Dieu Chau
Xem chi tiết
Nguyễn Hoàng Minh
16 tháng 12 2021 lúc 10:53

\(a,S=\dfrac{\left(2014+4\right)\left[\left(2014-4\right):3+1\right]}{2}=\dfrac{2018\cdot671}{2}=677039\\ b,\forall n\text{ lẻ }\Rightarrow n+2013\text{ chẵn }\Rightarrow n\left(n+2013\right)⋮2\left(1\right)\\ \forall n\text{ chẵn }\Rightarrow n\left(n+2013\right)⋮2\left(2\right)\\ \left(1\right)\left(2\right)\RightarrowĐpcm\\ c,M=\left(2+2^2+2^3+2^4\right)+...+\left(2^{17}+2^{18}+2^{19}+2^{10}\right)\\ M=2\left(1+2+2^2+2^3\right)+...+2^{16}\left(1+2+2^2+2^3\right)\\ M=\left(1+2+2^2+2^3\right)\left(2+...+2^{16}\right)=15\left(2+...+2^{16}\right)⋮15\)

Bình luận (1)
Nguyễn vũ kim ngân
Xem chi tiết
Vũ Thị Tuệ Linh
Xem chi tiết
Yen Nhi
18 tháng 3 2022 lúc 22:24

`Answer:`

1. \(S=\frac{1}{41}+\frac{1}{42}+...+\frac{1}{80}\)

\(\Rightarrow S=\left(\frac{1}{41}+\frac{1}{42}+...+\frac{1}{60}\right)+\left(\frac{1}{61}+...+\frac{1}{80}\right)\)

\(\Rightarrow S>\left(\frac{1}{60}+\frac{1}{60}+...+\frac{1}{60}\right)+\left(\frac{1}{80}+...+\frac{1}{80}\right)\)

\(\Rightarrow S>20.\frac{1}{60}+20.\frac{1}{80}\)

\(\Rightarrow S>\frac{1}{3}+\frac{1}{4}\)

\(\Rightarrow S>\frac{7}{12}\)

2. \(S=\frac{1}{2^2}+\frac{1}{3^2}+\frac{1}{4^2}+...+\frac{1}{2009^2}\)

Ta có:

 \(2^2< 1.2\Rightarrow\frac{1}{2^2}< \frac{1}{1.2}\)

\(3^2< 2.3\Rightarrow\frac{1}{3^2}< \frac{1}{2.3}\)

\(4^2< 3.4\Rightarrow\frac{1}{4^2}< \frac{1}{3.4}\)

...

\(2009^2< 2008.2009\Rightarrow\frac{1}{2009^2}< \frac{1}{2008.2009}\)

\(\Rightarrow S< \frac{1}{1.2}+\frac{1}{2.3}+\frac{1}{3.4}+...+\frac{1}{2008.2009}\)

\(\Rightarrow S< 1-\frac{1}{2}+\frac{1}{2}-\frac{1}{3}+\frac{1}{3}-\frac{1}{4}+...+\frac{1}{2008}-\frac{1}{2009}\)

\(\Rightarrow S< 1-\frac{1}{2009}< 1\)

\(\Rightarrow S< 1\)

3. \(\frac{3}{5.8}+\frac{11}{8.19}+\frac{12}{19.31}+\frac{70}{31.101}+\frac{99}{101.200}\)

\(=\frac{1}{5}-\frac{1}{8}+\frac{1}{8}-\frac{1}{19}+\frac{1}{19}-\frac{1}{31}+\frac{1}{31}-\frac{1}{101}+\frac{1}{101}-\frac{1}{200}\)

\(=\frac{1}{5}-\frac{1}{200}\)

\(=\frac{39}{200}\)

Bình luận (0)
 Khách vãng lai đã xóa